Question and Answers Forum

All Questions      Topic List

Limits Questions

Previous in All Question      Next in All Question      

Previous in Limits      Next in Limits      

Question Number 130843 by EDWIN88 last updated on 29/Jan/21

 Without L′Ho^� pital    lim_(x→1)  ((((x^3 +7))^(1/3)  − (√(x^2 +3)))/(x−1)) ?

$$\:{Without}\:{L}'{H}\hat {{o}pital}\: \\ $$$$\:\underset{{x}\rightarrow\mathrm{1}} {\mathrm{lim}}\:\frac{\sqrt[{\mathrm{3}}]{{x}^{\mathrm{3}} +\mathrm{7}}\:−\:\sqrt{{x}^{\mathrm{2}} +\mathrm{3}}}{{x}−\mathrm{1}}\:?\: \\ $$

Answered by mathmax by abdo last updated on 29/Jan/21

let f(x)=(((x^3 +7)^(1/3) −(x^2  +3)^(1/2) )/(x−1)) we do the changement x−1=t ⇒  f(x)=f(t+1) =((((t+1)^3  +7)^(1/3) −((t+1)^2  +3)^(1/2) )/t)  =(((t^3  +3t^2  +3t +8)^(1/3) −(t^2  +2t+4)^(1/2) )/t)  =((8^(1/3) (1+(1/8)(t^3  +3t^2  +3t))^(1/3) −4^(1/2) (1+(1/4)(t^2  +2t))^(1/2) )/t)( t→0) ⇒  f(t+1)∼((2{1+(1/(24))(t^3  +3t^2  +3t)}−2(1+(1/8)(t^2  +2t)))/t)  ⇒f(t+1)∼(((1/(12))(t^3  +3t^2  +3t)−(1/4)(t^2  +2t))/t)  ⇒f(t+1)∼(1/(12))(t^2  +3t+3)−(1/4)(t+2) ⇒  lim_(t→0) f(t+1) =(3/(12))−(2/4)=(1/4)−(1/2)=−(1/4) ⇒  lim_(x→1) f(x)=−(1/4)

$$\mathrm{let}\:\mathrm{f}\left(\mathrm{x}\right)=\frac{\left(\mathrm{x}^{\mathrm{3}} +\mathrm{7}\right)^{\frac{\mathrm{1}}{\mathrm{3}}} −\left(\mathrm{x}^{\mathrm{2}} \:+\mathrm{3}\right)^{\frac{\mathrm{1}}{\mathrm{2}}} }{\mathrm{x}−\mathrm{1}}\:\mathrm{we}\:\mathrm{do}\:\mathrm{the}\:\mathrm{changement}\:\mathrm{x}−\mathrm{1}=\mathrm{t}\:\Rightarrow \\ $$$$\mathrm{f}\left(\mathrm{x}\right)=\mathrm{f}\left(\mathrm{t}+\mathrm{1}\right)\:=\frac{\left(\left(\mathrm{t}+\mathrm{1}\right)^{\mathrm{3}} \:+\mathrm{7}\right)^{\frac{\mathrm{1}}{\mathrm{3}}} −\left(\left(\mathrm{t}+\mathrm{1}\right)^{\mathrm{2}} \:+\mathrm{3}\right)^{\frac{\mathrm{1}}{\mathrm{2}}} }{\mathrm{t}} \\ $$$$=\frac{\left(\mathrm{t}^{\mathrm{3}} \:+\mathrm{3t}^{\mathrm{2}} \:+\mathrm{3t}\:+\mathrm{8}\right)^{\frac{\mathrm{1}}{\mathrm{3}}} −\left(\mathrm{t}^{\mathrm{2}} \:+\mathrm{2t}+\mathrm{4}\right)^{\frac{\mathrm{1}}{\mathrm{2}}} }{\mathrm{t}} \\ $$$$=\frac{\mathrm{8}^{\frac{\mathrm{1}}{\mathrm{3}}} \left(\mathrm{1}+\frac{\mathrm{1}}{\mathrm{8}}\left(\mathrm{t}^{\mathrm{3}} \:+\mathrm{3t}^{\mathrm{2}} \:+\mathrm{3t}\right)\right)^{\frac{\mathrm{1}}{\mathrm{3}}} −\mathrm{4}^{\frac{\mathrm{1}}{\mathrm{2}}} \left(\mathrm{1}+\frac{\mathrm{1}}{\mathrm{4}}\left(\mathrm{t}^{\mathrm{2}} \:+\mathrm{2t}\right)\right)^{\frac{\mathrm{1}}{\mathrm{2}}} }{\mathrm{t}}\left(\:\mathrm{t}\rightarrow\mathrm{0}\right)\:\Rightarrow \\ $$$$\mathrm{f}\left(\mathrm{t}+\mathrm{1}\right)\sim\frac{\mathrm{2}\left\{\mathrm{1}+\frac{\mathrm{1}}{\mathrm{24}}\left(\mathrm{t}^{\mathrm{3}} \:+\mathrm{3t}^{\mathrm{2}} \:+\mathrm{3t}\right)\right\}−\mathrm{2}\left(\mathrm{1}+\frac{\mathrm{1}}{\mathrm{8}}\left(\mathrm{t}^{\mathrm{2}} \:+\mathrm{2t}\right)\right)}{\mathrm{t}} \\ $$$$\Rightarrow\mathrm{f}\left(\mathrm{t}+\mathrm{1}\right)\sim\frac{\frac{\mathrm{1}}{\mathrm{12}}\left(\mathrm{t}^{\mathrm{3}} \:+\mathrm{3t}^{\mathrm{2}} \:+\mathrm{3t}\right)−\frac{\mathrm{1}}{\mathrm{4}}\left(\mathrm{t}^{\mathrm{2}} \:+\mathrm{2t}\right)}{\mathrm{t}} \\ $$$$\Rightarrow\mathrm{f}\left(\mathrm{t}+\mathrm{1}\right)\sim\frac{\mathrm{1}}{\mathrm{12}}\left(\mathrm{t}^{\mathrm{2}} \:+\mathrm{3t}+\mathrm{3}\right)−\frac{\mathrm{1}}{\mathrm{4}}\left(\mathrm{t}+\mathrm{2}\right)\:\Rightarrow \\ $$$$\mathrm{lim}_{\mathrm{t}\rightarrow\mathrm{0}} \mathrm{f}\left(\mathrm{t}+\mathrm{1}\right)\:=\frac{\mathrm{3}}{\mathrm{12}}−\frac{\mathrm{2}}{\mathrm{4}}=\frac{\mathrm{1}}{\mathrm{4}}−\frac{\mathrm{1}}{\mathrm{2}}=−\frac{\mathrm{1}}{\mathrm{4}}\:\Rightarrow \\ $$$$\mathrm{lim}_{\mathrm{x}\rightarrow\mathrm{1}} \mathrm{f}\left(\mathrm{x}\right)=−\frac{\mathrm{1}}{\mathrm{4}} \\ $$

Commented by EDWIN88 last updated on 30/Jan/21

yes..thank you

$${yes}..{thank}\:{you} \\ $$

Commented by mathmax by abdo last updated on 30/Jan/21

you are welcome

$$\mathrm{you}\:\mathrm{are}\:\mathrm{welcome} \\ $$

Terms of Service

Privacy Policy

Contact: info@tinkutara.com